AAMC CBT6 and 6R OFFICIAL Q&A

This forum made possible through the generous support of SDN members, donors, and sponsors. Thank you.

Vihsadas

No summer
Moderator Emeritus
Lifetime Donor
10+ Year Member
5+ Year Member
15+ Year Member
Joined
Oct 17, 2007
Messages
5,474
Reaction score
56
This is the official Q&A thread for AAMC CBT6 and 6R.

Please post ONLY questions pertaining to AAMC CBT6 and 6R.
Out of respect for people who may not have completed the other exams, do not post questions or material from any other AAMC exam.

Please see this thread for the rules of order before you post.

Good luck on your MCAT!

Members don't see this ad.
 
Wait are they the same thing? does 6R just mean its on paper??
 
Wait are they the same thing? does 6R just mean its on paper??

6CBT is the same as 6R but is missing some of the passages and discretes. AAMC just cut down the size of the older paper exams so they would be relevant as CBT practice exams.
 
For the 6r paper test on the PS section I had a question about #49. Is this based on pv=nrt where since compound 2 is larger than 1 it has a smaller number of moles making the graph of the compound lower? Thanks.
 
Members don't see this ad :)
did anyone else find this passage particularly difficult? #35 seem like it could be a, b, or c...
ideas?
 
Yea that answer explanation isn't very great..It just says the sun doesn't reflect gas or something.

Can anyone explain #52. Even after deriving all forms of P= IV I feel like V and I are proportional to each other so I don't really understand the AAMC explanation of using high voltage and low current to reduce heat loss....
 
Yea that answer explanation isn't very great..It just says the sun doesn't reflect gas or something.

Can anyone explain #52. Even after deriving all forms of P= IV I feel like V and I are proportional to each other so I don't really understand the AAMC explanation of using high voltage and low current to reduce heat loss....

Pout = Pin - Pr (lost from resistance)
Pout = IV - I^2R

The current will not change in a series resistor, so you are dealing with V and R. If you increase the Voltage then the Power in will increase and then the Pout will be greater. Hope this helps.
 
Pout = Pin - Pr (lost from resistance)
Pout = IV - I^2R

The current will not change in a series resistor, so you are dealing with V and R. If you increase the Voltage then the Power in will increase and then the Pout will be greater. Hope this helps.

The only problem with this explanation (and others as well) is that according to Ohm's law, if V increases, then so does I. While all circuit elements in series share the same current, it does not mean that the current is constant with a change in voltage.

If it were true that raising voltage did not change current, then it would be raising resistance, which is not possible for separate devices.

I tend to agree that this question cannot be answered. I know there are answers out there that were written after the fact (once AAMC stated what they wanted the answer to be), but they all have inconsistencies, including the ones present in our (TBRs) explanation.

I guess this just proves that it's a subjective exam to some extent. What I tell my students for this question is that D is correct because they want you to pick D.

For anyone who is interested, the answer explanation on the Berkeley Review's version of the answer key is as follows:

Choice D is correct.
To be perfectly honest, I have absolutely no idea how they arrived at a correct answer to this question. Based on the fact that 24% of test-takers got this question correct and random guessing would generate 25% correct, it appears that all of us have no idea what the test-writer was thinking here. Choices A and B have similarity in that currents of higher density and higher currents are saying something similar. You cannot have two “best” answers, so choices A and B exclude one another. The question is reduced to an intuitive guess between reducing the amount of insulation needed versus reducing the heat loss. It would seem that reducing heat loss would be a good idea when sending current through power lines, because all of the energy dissipated as heat would be wasted. Reduced insulation doesn’t seem likely, because you have a greater current with a “stepped-up” voltage, which would require more insulation. Choice D is the best of four bad choices.
 
Why does dust reflect better than gas?

Dust, being a solid, has a surface. As such, it can reflect. If you think about a beam of light in a dark room you see it illuminating all of the dust particles hovering in the air, but it doesn't illuminate the nitrogen gas or oxygen gas, etc...

Basically, it just comes down to needing a surface to have a reflection.
 
Dust, being a solid, has a surface. As such, it can reflect. If you think about a beam of light in a dark room you see it illuminating all of the dust particles hovering in the air, but it doesn't illuminate the nitrogen gas or oxygen gas, etc...

Basically, it just comes down to needing a surface to have a reflection.

Thank you!
 
In the solutions, all it says is the passage only talks about the lymphatic system but I'm lost to what exactly should have tipped me off about lymphatic system in the passage. Help please. :)
 
The only problem with this explanation (and others as well) is that according to Ohm's law, if V increases, then so does I. While all circuit elements in series share the same current, it does not mean that the current is constant with a change in voltage.

If it were true that raising voltage did not change current, then it would be raising resistance, which is not possible for separate devices.

I tend to agree that this question cannot be answered. I know there are answers out there that were written after the fact (once AAMC stated what they wanted the answer to be), but they all have inconsistencies, including the ones present in our (TBRs) explanation.

I guess this just proves that it's a subjective exam to some extent. What I tell my students for this question is that D is correct because they want you to pick D.

For anyone who is interested, the answer explanation on the Berkeley Review's version of the answer key is as follows:

thank you for this... i remember spending way to much time on this question...
 
Members don't see this ad :)
Dust, being a solid, has a surface. As such, it can reflect. If you think about a beam of light in a dark room you see it illuminating all of the dust particles hovering in the air, but it doesn't illuminate the nitrogen gas or oxygen gas, etc...

Basically, it just comes down to needing a surface to have a reflection.

couldn't the gas particles emit light via the photoelectric effect?

maybe im thinking too deeply on this...
 
couldn't the gas particles emit light via the photoelectric effect?

maybe im thinking too deeply on this...

In the simplest perspective, the photoelectric effect does just the opposite. Light of a high enough energy is absorbed to overcome the work function (similar to the ionization energy, but for a solid) and an electron is ejected. But the photoelectric effect is more likely to occur with a metal than a nonmetal, and most gases are nonmetals and nonmetal oxides.

It does bring up the fact that much of the upper atmosphere is ionized (resulting in a plasma).
 
cool thanks..

i got question about #46

i understand that the different element needs to have more protons than Li, but whose to say this different element needs to have more neutrons? If the element had more protons and few protons, couldn't the q/m ratio still be greater than Li?
 
got another one (sorry just took this today...)

i understand why the Z conformation is possible given the data, but what excludes the A conformation... doesn't the observed helical twist fall well within its range?
 
AAMC 6CBT - BioSci Section - Question #141

"Geminal dihalide reacts with two equiv of alkoxide ---> product??? ..."

The product should be a Ketal and not an Acetal, right?
AAMC says the answer is an Acetal... I'm kind of lost.

Tried searching this thread but couldn't find anything, so I'm guessing I'm wrong?
 
If Steve’s blood pressure had increased significantly more than Joe’s increased (other factors being equal) while they worked, what difference in their urinary system function would be expected? A
) Joe’s glomerular filtration rate would increase more than Steve’s would. B
) Steve’s reabsorption rate of glomerular filtrate by the peritubular capillaries would be lower than Joe’s would be. C
) Steve’s reabsorption rate per milliliter of glomerular filtrate by the peritubular capillaries would be higher than Joe’s would be. D
) Steve’s glomerular filtration rate would increase more than Joe’s would.

arghhh... so it answer was D because they were referring to "arterial blood pressure" and therefore hydrostatic pressure on the glomerulous would increase the rate of filtration.. I assumed they were talking about BP in general.. and thought it had to do with reabsorption...

can anyone tell me HOW I WAS SUPPOSE TO KNOW WHAT KIND OF BP THEY WERE REFERRING TO?
 
If Steve's blood pressure had increased significantly more than Joe's increased (other factors being equal) while they worked, what difference in their urinary system function would be expected? A
) Joe's glomerular filtration rate would increase more than Steve's would. B
) Steve's reabsorption rate of glomerular filtrate by the peritubular capillaries would be lower than Joe's would be. C
) Steve's reabsorption rate per milliliter of glomerular filtrate by the peritubular capillaries would be higher than Joe's would be. D
) Steve's glomerular filtration rate would increase more than Joe's would.

arghhh... so it answer was D because they were referring to "arterial blood pressure" and therefore hydrostatic pressure on the glomerulous would increase the rate of filtration.. I assumed they were talking about BP in general.. and thought it had to do with reabsorption...

can anyone tell me HOW I WAS SUPPOSE TO KNOW WHAT KIND OF BP THEY WERE REFERRING TO?

well when you speak of taking someones blood pressure you always speak of the arteries, never the veins. the blood pressure in the veins is too low. that shouldnt have caused you troulbe in finding the answer though. if you have a higher blood pressure, there is going to be more pressure to push fluid out, which will increase the level of filtration in the glomerulus. this is what choice D is saying.
 
Hello RHC,
Thanks for visiting the MCAT forum. I've merged this thread with the 'AAMC 6R and 6CBT' official Q&A thread. In the future, when you have questions about an AAMC exam, please only post the question in the thread that is designated for the specific AAMC exam that you have questions about.

Please see the following thread for the rules of order when posting about AAMC exams:
http://forums.studentdoctor.net/showthread.php?t=516243

Thanks, and I hope that you find the help that you need to do well on your MCAT!
 
I only have the R version of that exam, but I think that you mean the question that talks about a GPS satellite that sends a signal to earth in .7 seconds.

Think about the following situation.

We are astronauts in a space station where there is no gravity:
- I am positioned to your right at a 0 deg angle to the shuttle's cockpit and I throw you a ball at you a velocity 'x' that takes .7s to get to you.
- Then I float to the top of the shuttle, where I am now positioned at a 40 deg angle relative to the cockpit of the shuttle. I again throw you a ball with the same speed. It again takes .7s to get to you.

Knowing that,
Is the distance between me and you in the first situation and in the second situation different?

Now think about yourself as the receiver and me as the satellite.
 
I only have the R version of that exam, but I think that you mean the question that talks about a GPS satellite that sends a signal to earth in .7 seconds.

Think about the following situation.

We are astronauts in a space station where there is no gravity:
- I am positioned to your right at a 0 deg angle to the shuttle's cockpit and I throw you a ball at you a velocity 'x' that takes .7s to get to you.
- Then I float to the top of the shuttle, where I am now positioned at a 40 deg angle relative to the cockpit of the shuttle. I again throw you a ball with the same speed. It again takes .7s to get to you.

Knowing that,
Is the distance between me and you in the first situation and in the second situation different?

Now think about yourself as the receiver and me as the satellite.

The distance isn't different, but I guess I thought the initial velocity would be affected by the angle. This doesn't pertain to the speed of light?
 
Yeah still really dont understand why you dont take into consideration an angle for that problem you are talking about..
 
v = d/t

the laser (going off memory so it could be a non-laser) travels 3.0x10^8 meters per second if you point it up, down, or to the side so the angle doesn't matter. Now if it takes (blank) amount of time then the distance is 2d = vt (if its back and forth, which I think it was)

The reason you don't use angles is because the receiver is in a straight path to the satellite. Although it is at an angle to the earth's horizon, the receiver must also be at an angle to the earth horizon to capture it (for the simplistic explanation).
 
Thanks.


Okay this might be kind of dumb, but how can you ASSUME for #21 in PS that the B.P. will be higher than 10 degrees C. I put B, less than 10 degrees C, because I thought over 10 was a little much, clearly I knew the additional H bonds had to increase it, but how do we KNOW for sure it is A and not B (AAMC's explanation does not help).

thanks
 
Thanks.


Okay this might be kind of dumb, but how can you ASSUME for #21 in PS that the B.P. will be higher than 10 degrees C. I put B, less than 10 degrees C, because I thought over 10 was a little much, clearly I knew the additional H bonds had to increase it, but how do we KNOW for sure it is A and not B (AAMC's explanation does not help).

thanks
yeah...it was kinda stupid. the way i interpreted the answer choices:
a. much higher BP
b. a little higher BP
c. a little lower BP
d. much lower BP
seeing as it has the opportunity to form many strong hydrogen bonds, A would be the best answer. but you would never be able to figure out by how much.
 
PS #15 (question about copper getting reduced, galvanic cell)

the explanation states: "The cell potential can be found by subtracting the standard reduction potential for the substance being oxidized from that of the substance being reduced."

So I subtract - .13 (Pb) - .52 = -.65 BUT this is negative and the explanation states that the net reaction must be positive in order for the reaction to occur spontaneously???

Confused!
 
got another one (sorry just took this today...)

i understand why the Z conformation is possible given the data, but what excludes the A conformation... doesn't the observed helical twist fall well within its range?
Sorry for the delayed response. But I think I know what you're referring to and the answer is that the other two angles are too large to fall within the 30 degree angle of the 12 amino acids that form the 360 helix. 360/12 = 30 degrees, only Z falls in that range.


and for maluskeeter
There is one thing you should know about reduction potentials that would've helped you nail this question in a heartbeat. Copper has a positive reduction, which means it would have a tendency of being reduced. Zinc and Lead both have negative potentials, which in the scheme of things, have a better potential of being oxidized (flip the equation and you get the positive E). Silver, like Copper, has a positive E which means it, too, would want to get reduced.

All E measures is the tendency of an element to be reduced. If it has a larger positive value for the reduction potential, then it would spontaneously reduce into Copper instead of oxidizing to a Copper ion(The reverse).
G = -nFE. From there, we can find just how reluctant an element is to be reduced. A more positive E = a more negative deltaG - spontaneous.

Taking that into account, what the question basically asks, is for you to find which of the 3 elements would work with Copper to help Copper be reduced while they become oxidized. Subtracting their values from Copper (which upon noticing, you did the reverse) basically gives X = Cu - Y. With Y being the element's E and X being the total. If X yields a positive value, then you have a working galvonic cell which would have Copper being reduced and the other element being oxidized. Silver wouldn't work because it would yield a negative value and instead favors the oxidation of Copper and the reduction of Silver ions.

A positive E - favors reduction. A negative E - favors oxidation
 
In the solutions, all it says is the passage only talks about the lymphatic system but I'm lost to what exactly should have tipped me off about lymphatic system in the passage. Help please. :)

This is how I reasoned it: TSS results in high fever, hypotension, and rash, according to the passage. Hypotension means TSS is affecting the circulatory system, rashes mean it's affecting the skin, so which system out of the four answer choices if affected when the patient has a high fever? Since high fevers indicate an immune response and the lymphatic system is involved in fighting disease (production of lymphocytes), I thought this was the best choice.
 
hey, how would you guys say AAMC6 compares to the other practice exams? I've been scoring in the high 20's for AAMC3,5,7 but somehow i managed a 33 (11 PS, 11 VR, 11 BS). This was the first time i broke a 30 (so excited:D). Oh yeah, i usually get 8's on verbal sections so i dont know what happened here. I hope this wasn't just a fluke. Thanks!
 
PS #26
decreasing g by 6 increases time of total flight by 6. I see how to get this answer by using V=Vinitial + at, but if i use d= Vt -1/2at^2, i get that t is increased by square root 6. im sure im just making a stupid math mistake. can someone explain this to me? thanks!
 
We're not giving any information on the distance that the object travels. I made the same error; since we're just talking about V,a, and t, we need to use V = at. Since a decreases by a factor of 6, t increases by a factor of 6.
 
Question 1:

Scientists are most likely to find which compound in table 1 in an alkaline lake:
a) NaCl
b) CH3OH
c) H2SO4
d) KOH

I picked B because all of the other compounds are completely soluble in H20. Only B would exist as a compound. They say the right answer is D because its basic. I call BS.
 
Question 1:

Scientists are most likely to find which compound in table 1 in an alkaline lake:
a) NaCl
b) CH3OH
c) H2SO4
d) KOH

I picked B because all of the other compounds are completely soluble in H20. Only B would exist as a compound. They say the right answer is D because its basic. I call BS.
Yeah, you over-thought the answer. The passages was talking about alkaline lakes or something like that, then asked which of the following would be likely found in an alkaline lake. The best answer will be the one that most efficiently ionizes into a basic molecule, i.e. KOH --> K+ + -OH. But, I see where you came from though. The questions was just posed in a strange fashion, IMHO.
 
Last edited:
This is more of a complaint than anything else, but, I really thought this was a stupid question. I was confident that this question had something to do with Joe's scared body leading to overheating etc, etc, etc, not that Steve was more frightened than Joe. *sigh* Anybody feeling me here? There were a few really stupid questions on AAMC 6, IMO. But this one and the last one in the PS section (outlined earlier in the thread) really ticked me off.

If the heart rates of Steve and Joe were monitored when they entered the burning warehouse to fight the fire, one would expect:

A) Steve's heart rate to increase more, because of activation of the sympathetic division of the autonomic nervous system.
B) Joe's heart rate to increase more, because of activation of the sympathetic division of the autonomic nervous system.
C) Joe's heart rate to increase more, because of activation of the parasympathetic division of the autonomic nervous system.
D) both heart rates to increase, because of activation of the parasympathetic division of the autonomic nervous system.
 
In the dirty-snowball model, does the perturbing force on the comet due to sublimation act in any preferred direction?

A) No, because the nucleus tends to have a roughly spherical surface

B) No, because the sun radiates with equal intensity in all directions

C) Yes, more or less outward from the sun because of shadowing effects
Sunlight is absorbed on one side of the comet. The sublimating gases are preferentially ejected toward the Sun because the solids block the other direction. The force on the comet is away from the Sun. Choice C states this conclusion.


D) Yes, more or less toward the sun because of the temperature gradient

I thought the answer would be B. Where in the passage does it say that the "solids block the other sirection"??
 
Electric power for transmission over long distances is “stepped up” to a very high voltage in order:

A) to produce currents of higher density.

B) to produce higher currents in the transmission wires.

C) to make less insulation necessary.

D) to cut down the heat loss in the transmission wires.
Power transmitted to a load at the user’s end of an electrical line is the product of the current I times the voltage V at the user’s end. The power company has the option of using a low voltage or a high voltage for a given amount of power delivered. Using Ohm’s law (I = V/R), the power delivered can also be written as P = V2/R. The power lost in the transmission line is I2R. Since the resistance R is fixed by the material and dimensions of the line, we see that using a high voltage at a lower current maximizes the power to the user and minimizes the heat loss in the transmission line, choice D.

I am not satisfied with this answer. There is no difference between the formulas I2R and V2/R. Does any one understand this explanation? What exactly is the formula for the heat loss?
 
It was a horrible question, don't worry about it, honestly. The correct answer rate is 24%--that's right, its essentially "random" guessing. From what I read this was a largely unjustified question and answer and anybody who got it right was lucky.
 
In the dirty-snowball model, does the perturbing force on the comet due to sublimation act in any preferred direction?

A) No, because the nucleus tends to have a roughly spherical surface

B) No, because the sun radiates with equal intensity in all directions

C) Yes, more or less outward from the sun because of shadowing effects
Sunlight is absorbed on one side of the comet. The sublimating gases are preferentially ejected toward the Sun because the solids block the other direction. The force on the comet is away from the Sun. Choice C states this conclusion.


D) Yes, more or less toward the sun because of the temperature gradient

I thought the answer would be B. Where in the passage does it say that the "solids block the other sirection"??
The shaded side won't be as perturbed, right? So, the unshaded side will lead to an "explosion" so to speak, and it will produce a force on the meteorite outward away from the sun b/c of the fact that the explosion is occurring on the sun side and relatively little is happening on the shaded side.
 
If all genotypes are equally fit and if there are no genetic modifiers of the sex ratio trait, what will be the ultimate fate of a population in which 50% of the x chromosomes are currently xi and 50% are xs.
A. Extinction
B. Stable population size, with predominance of females
C. Stable population size with all individuals producing a 50 50 sex ratio
D. Stable population size, with some individuals producing an excess of females and some producing an excess of males.

My answer was B. The answer key says A, but isn't extinction a clear impossibility given that ALL GENOTYPES are equally fit? This is just confusing. :thumbdown:thumbdown:thumbdown:

also, did anyone else think this aamc practice test was loaded with gotcha type questions in PS and BS, typically the types of questions you find on the VR section?
 
Electric power for transmission over long distances is "stepped up" to a very high voltage in order:

A) to produce currents of higher density.

B) to produce higher currents in the transmission wires.

C) to make less insulation necessary.

D) to cut down the heat loss in the transmission wires.
Power transmitted to a load at the user's end of an electrical line is the product of the current I times the voltage V at the user's end. The power company has the option of using a low voltage or a high voltage for a given amount of power delivered. Using Ohm's law (I = V/R), the power delivered can also be written as P = V2/R. The power lost in the transmission line is I2R. Since the resistance R is fixed by the material and dimensions of the line, we see that using a high voltage at a lower current maximizes the power to the user and minimizes the heat loss in the transmission line, choice D.

I am not satisfied with this answer. There is no difference between the formulas I2R and V2/R. Does any one understand this explanation? What exactly is the formula for the heat loss?

I think this problem is using the Joule-Lenz Law, which describes the amount of heat dissappated by a conductor, where Q=(I^2)(R)(t), where I is the current, R is the resistance in the conductor and t is the time. If you were to decrease the current by 3 times, by ramping up the voltage, then your heat loss would decrease by a factor of 9. I'm pretty surprised that this would show up in a question though. I barely remembered this from a circuits class and had to look it up!
 
Top